Znaleziono 1703 wyniki

autor: timon92
3 sty 2023, o 12:05
Forum: Granica i ciągłość funkcji
Temat: Granica z pierwiastkami
Odpowiedzi: 6
Odsłony: 1062

Re: Granica z pierwiastkami

a4karo pisze: 11 lip 2022, o 14:11 ... jest wypukła jako odwrotność funkcji wklęsłej ...
na ogół to tak nie jest: funkcja \(x \mapsto -x\) jest wklęsła na \((0,\infty)\), a jej odwrotność nie jest wypukła
autor: timon92
3 sty 2023, o 09:22
Forum: Kółko matematyczne
Temat: [Nierówności] Dwie zmienne rzeczywiste i stała
Odpowiedzi: 3
Odsłony: 414

Re: [Nierówności] Dwie zmienne rzeczywiste i stała

odpowiedź to
Timonek pisze:już mi się nie chce tego wklepywać do wolframa
autor: timon92
28 gru 2022, o 11:35
Forum: Geometria trójkąta
Temat: Ortocentrum i równoległobok
Odpowiedzi: 2
Odsłony: 294

Re: Ortocentrum i równoległobok

na kątach wychodzi, że punkty \(F, O_1, O_2, O_3\) leżą na okręgu jest jasne, że \(A\) jest symetryczny do \(F\) względem \(O_1O_3\), a \(B\) jest symetryczny do \(F\) względem \(O_1O_2\) w takim razie \(AB\) jest prostą Steinera punktu \(F\) względem trójkąta \(O_1O_2O_3\), więc ortocentrum \(O_1O_...
autor: timon92
27 gru 2022, o 00:32
Forum: Zadania "z treścią"
Temat: Różnica będąca kwadratem
Odpowiedzi: 8
Odsłony: 962

Re: Różnica będąca kwadratem

gołym okiem widać, że ten bełkot (jak i pozostałe posty Pana Profesora) są wygenerowane przez chatgpt
autor: timon92
23 gru 2022, o 17:32
Forum: Własności i granice ciągów
Temat: Liczba Eulera do wyznaczania granicy?
Odpowiedzi: 43
Odsłony: 2603

Re: Liczba Eulera do wyznaczania granicy?

3a174ad9764fefcb pisze: 23 gru 2022, o 12:12 nie widzę sensu w definiowaniu zbieżności na przestrzeni antydyskretnej.
granicę ciągu można zdefiniować w dowolnej przestrzeni topologicznej tak jak pisał a4karo

ciąg może mieć wiele granic, ale jeśli przestrzeń spełnia aksjomat \(T_2\) to granica jest jednoznaczna (oczywiście o ile istnieje)
autor: timon92
20 gru 2022, o 13:46
Forum: Podzielność
Temat: Podzielność przez 2017
Odpowiedzi: 7
Odsłony: 682

Re: Podzielność przez 2017

\(\frac 1k + \frac{1}{2017-k} = \frac{2017}{k(2017-k)}\), więc Twoje wyrażenie równa się \(2017 \cdot \left(\frac{1}{1\cdot 2016} + \frac{1}{2\cdot 2015} + \ldots + \frac{1}{1008 \cdot 1009}\right)\cdot 2 \cdot 3 \cdot \ldots \cdot 2016\)
autor: timon92
19 gru 2022, o 11:30
Forum: Przekształcenia algebraiczne
Temat: Nierówność - liczba zmiennych
Odpowiedzi: 6
Odsłony: 955

Re: Nierówność - liczba zmiennych

też mi wychodzi, że najmniejszym możliwym \(n\) jest \(18\) bez straty ogólności wszystkie zmienne są dodatnie, bo jeśli mam jakieś \(x_1,x_2,\ldots,x_n\), które to spełniają, to mogę pominąć niedodatnie liczby i dostanę \(m\)-tkę, która też to spełnia i \(m \le n\) w pierwszej nierówności mogę bez ...
autor: timon92
18 gru 2022, o 13:53
Forum: Kółko matematyczne
Temat: [Nierówności] Dwie zmienne rzeczywiste i stała
Odpowiedzi: 3
Odsłony: 414

Re: [Nierówności] Dwie zmienne rzeczywiste i stała

\(k=-\frac 12\) działa, od teraz \(k\neq -\frac 12\) przez skalowanie można ograniczyć się do przypadku gdy \(x+y=1\) lewa strona to \(\frac{1}{x^2}+\frac{1}{y^2}+k(2k+1) = \frac{x^2+y^2}{x^2y^2}+k(2k+1)=\frac{1-2xy}{(xy)^2}+k(2k+1)\), a prawa \(3(2k-1)\sqrt[3]{\frac{2k+1}{16(xy)^2}}\) podstawmy now...
autor: timon92
2 lis 2022, o 21:04
Forum: Planimetria
Temat: Trójkąt na szachownicy
Odpowiedzi: 11
Odsłony: 543

Re: Trójkąt na szachownicy

jakiś konkretny trójkąt czy dowolny?
autor: timon92
25 paź 2022, o 19:08
Forum: Kółko matematyczne
Temat: [Rozgrzewka OM][MIX][Planimetria] Planimetria
Odpowiedzi: 211
Odsłony: 80677

Re: [Rozgrzewka OM][MIX][Planimetria] Planimetria

jak to nie leżą?
autor: timon92
25 paź 2022, o 00:07
Forum: Kółko matematyczne
Temat: [Rozgrzewka OM][MIX][Planimetria] Planimetria
Odpowiedzi: 211
Odsłony: 80677

Re: [Rozgrzewka OM][MIX][Planimetria] Planimetria

dobrze przepisałeś treść?

te środki leżą na linii środkowej trójkąta \(ABC\) równoległej do boku \(AB\)
autor: timon92
22 paź 2022, o 00:28
Forum: Zbiory. Teoria mnogości
Temat: Rozkłady zbiorów
Odpowiedzi: 34
Odsłony: 3555

Re: Rozkłady zbiorów

a jaka byłaby odpowiedź, gdybyśmy chcieli przestrzeń trójwymiarową z wyjętą prostą podzielić na domknięte półpłaszczyzny?
autor: timon92
13 paź 2022, o 18:17
Forum: Kombinatoryka i matematyka dyskretna
Temat: Kolorowe liczby
Odpowiedzi: 3
Odsłony: 204

Re: Kolorowe liczby

@up zawsze będą istnieć dowolnie długie (ale skończone) monochromatyczne ciągi arytmetyczne --- zgooglaj sobie twierdzenie van der Waerdena
autor: timon92
16 wrz 2022, o 22:24
Forum: Geometria trójkąta
Temat: różnica kątów trójkącie
Odpowiedzi: 10
Odsłony: 893

Re: różnica kątów trójkącie

możemy odwrócić kota ogonem i dowodzić czegoś takiego: dany jest czworokąt \(ADIE\), w którym \(\angle IED = 18^\circ, \angle DEA = 30^\circ, \angle ADE = 54^\circ, \angle EDI = 24^\circ\); wykazać, że \(AI\) jest dwusieczną \(EAD\) zbudujmy trójkąt równoboczny \(ADF\) (\(F\) leży po tej samej stron...
autor: timon92
16 wrz 2022, o 01:19
Forum: Kółko matematyczne
Temat: [geometria] trójkąty i okręgi
Odpowiedzi: 3
Odsłony: 450

Re: [geometria] trójkąty i okręgi

bo brakło dżemu

jeśli ten czwarty punkt na okręgu opisanym na \(BDE\) oznaczymy przez \(F\), to \(\angle BDE = \angle BFE = \angle CAE\), więc czworokąt \(ACDE\) jest wpisany w okrąg

teraz już widać, że trójkąt \(ACD\) jest równoboczny, a teza zadania to nieskończenie znany fakt